K
Khách

Hãy nhập câu hỏi của bạn vào đây, nếu là tài khoản VIP, bạn sẽ được ưu tiên trả lời.

23 tháng 8 2020

Áp dụng bất đẳng thức Bunyakovsky ta được:          \(\left(ab+bc+ca+1\right)\left(\frac{a}{b}+\frac{b}{c}+\frac{c}{a}+1\right)\ge\left(a+b+c+1\right)^2\)\(\left(ab+bc+ca+1\right)\left(\frac{b}{a}+\frac{c}{b}+\frac{a}{c}+1\right)\ge\left(b+c+a+1\right)^2\)

Cộng theo vế hai bất đẳng thức này ta được \(\left(ab+bc+ca+1\right)\frac{\left(a+b\right)\left(b+c\right)\left(c+a\right)}{abc}\ge2\left(a+b+c+1\right)^2\)hay \(\frac{ab+bc+ca+1}{\left(a+b+c+1\right)^2}\ge\frac{2abc}{\left(a+b\right)\left(b+c\right)\left(c+a\right)}\)

Đến đây, ta quy bất đẳng thức cần chứng minh về dạng:\(\frac{2abc}{\left(a+b\right)\left(b+c\right)\left(c+a\right)}+\frac{3}{8}\sqrt[3]{\frac{\left(a+b\right)\left(b+c\right)\left(c+a\right)}{abc}}\ge1\)

Áp dụng bất đẳng thức Cauchy ta được \(\frac{2abc}{\left(a+b\right)\left(b+c\right)\left(c+a\right)}+\frac{1}{8}\sqrt[3]{\frac{\left(a+b\right)\left(b+c\right)\left(c+a\right)}{abc}}\)\(\ge2\sqrt{\frac{2abc}{\left(a+b\right)\left(b+c\right)\left(c+a\right)}.\frac{1}{8}\sqrt[3]{\frac{\left(a+b\right)\left(b+c\right)\left(c+a\right)}{abc}}}\)\(=\sqrt{\sqrt[3]{\frac{a^2b^2c^2}{\left(a+b\right)^2\left(b+c\right)^2\left(c+a\right)^2}}}=\sqrt[3]{\frac{abc}{\left(a+b\right)\left(b+c\right)\left(c+a\right)}}\)(*)

Cũng theo bất đẳng thức Cauchy ta được \(\sqrt[3]{\frac{abc}{\left(a+b\right)\left(b+c\right)\left(c+a\right)}}+\frac{1}{4}\sqrt[3]{\frac{\left(a+b\right)\left(b+c\right)\left(c+a\right)}{abc}}\ge2\sqrt{\frac{1}{4}}=1\)(**)

Từ (*) và (**) suy ra được \(\frac{2abc}{\left(a+b\right)\left(b+c\right)\left(c+a\right)}+\frac{3}{8}\sqrt[3]{\frac{\left(a+b\right)\left(b+c\right)\left(c+a\right)}{abc}}\ge1\)

Vậy bất đẳng thức được chứng minh

Đẳng thức xảy ra a = b = c = 1

23 tháng 8 2017

Bài này làm hoài :v

Áp dụng BĐT Cauchy-Schwarz và BĐT AM-GM ta có:

\(VT=\frac{b^2c^2}{ab+ac}+\frac{a^2c^2}{ab+bc}+\frac{a^2b^2}{ac+bc}\)

\(\ge\frac{\left(ab+bc+ca\right)^2}{2\left(ab+bc+ca\right)}=\frac{ab+bc+ca}{2}\)

\(\ge\frac{3\sqrt[3]{\left(abc\right)^2}}{2}=\frac{3}{2}=VP\)

Khi a=b=c=1

2 tháng 9 2020

Đặt \(\left\{a;b;c\right\}\rightarrow\left\{\frac{1}{x};\frac{1}{y};\frac{1}{z}\right\}\)Khi đó : \(\frac{1}{x}.\frac{1}{y}.\frac{1}{z}=\frac{1}{x.y.z}=a.b.c=1< =>x.y.z=1\)

\(BĐT< =>\frac{1}{\left(\frac{1}{x}\right)^3\left(\frac{1}{y}+\frac{1}{z}\right)}+\frac{1}{\left(\frac{1}{y}\right)^3\left(\frac{1}{y}+\frac{1}{x}\right)}+\frac{1}{\left(\frac{1}{z}\right)^3\left(\frac{1}{x}+\frac{1}{y}\right)}\ge\frac{3}{2}\)

\(< =>\frac{x^3yz}{y+z}+\frac{y^3xz}{z+x}+\frac{z^3xy}{x+y}\ge\frac{3}{2}\)\(< =>\frac{x^2}{y+z}+\frac{y^2}{x+z}+\frac{z^2}{x+y}\ge\frac{3}{2}\)(*)

Ta chỉ cần chỉ ra bất đẳng thức (*) đúng thì bài toán được giải quyết , thật vậy :

Theo bất đẳng thức Bunhiacopxki dạng phân thức :

\(\frac{x^2}{y+z}+\frac{y^2}{z+x}+\frac{z^2}{x+y}\ge\frac{\left(x+y+z\right)^2}{y+z+z+x+x+y}=\frac{\left(x+y+z\right)^2}{2\left(x+y+z\right)}=\frac{x+y+z}{2}\) (**)

Sử dụng bất đẳng thức AM-GM ta có : 

\(x+y+z\ge3\sqrt[3]{xyz}=3\sqrt[3]{1}=3\)Tương đương \(\frac{x+y+z}{2}\ge\frac{3}{2}\)(***)

Từ (**) và (***) ta được \(\frac{x^2}{z+y}+\frac{y^2}{x+z}+\frac{z^2}{x+y}\ge\frac{x+y+z}{2}\ge\frac{3}{2}\)

Suy ra bất đẳng thức (*) đúng . Nên ta có điều phải chứng minh !

Dấu "=" xảy ra khi và chỉ khi \(x=y=z=1< =>a=b=c=1\)

7 tháng 5 2020

\(\Leftrightarrow\Sigma\sqrt{\frac{3a^3}{\left[5a^2+\left(b+c\right)^2\right]\left(a+b+c\right)}}\le1\)

Theo Am-GM: \(VT=\Sigma\sqrt{\frac{3a^2}{5a^2+\left(b+c\right)^2}.\frac{a}{a+b+c}}\le\Sigma\frac{3a^2}{2\left(5a^2+\left(b+c\right)^2\right)}+\frac{1}{2}\)

Như vậy nó là đủ để chứng minh rằng: \(\Sigma\frac{3a^2}{5a^2+\left(b+c\right)^2}\le1\)

Giả sử \(c=min\left\{a,b,c\right\}\) nó tương đương:

$$2\, \left( a-b \right) ^{2} \left( 3\,c+a+b \right) \left( -c+a+b
 \right) \left( {a}^{2}+2\,ab+{b}^{2}+5\,{c}^{2} \right) +2\,c
 \left( a-c \right) \left( b-c \right) \left( 3\,{a}^{3}+9\,{a}^{2}b
+17\,c{a}^{2}+9\,a{b}^{2}-20\,abc+3\,{c}^{2}a+3\,{b}^{3}+17\,c{b}^{2}+
3\,{c}^{2}b+{c}^{3} \right) \geqq 0$$

(Gõ Latex, không hiện thì vô thống kê hỏi đáp xem)

Đây là điều hiển nhiên/

PS: Bài này quan trọng là ý tưởng phá căn thôi chứ không có gì khó. Lúc đầu UCT bất đẳng thức cuối cho đẹp nhưng phải xét các TH mệt lắm, chưa rành nên không làm cách đó:D

7 tháng 5 2020

Chứng minh: \(\Sigma\frac{3a^2}{5a^2+\left(b+c\right)^2}\le1\), cách 2:

Đổi biến sang pqr: (Vô thống kê hỏi đáp xem nếu olm không hiện Latex)

Nếu \(p^2\le4q\) ta cần:

$$2/9\,p \left( 19\,{p}^{2}-36\,q \right) \left( {p}^{3}-4\,qp+9\,r
 \right) -4/9\, \left( {p}^{2}-3\,q \right) \left( {p}^{2}-4\,q
 \right) \left( 5\,{p}^{2}-3\,q \right) \geqq 0$$

(Hiển nhiên)

Nếu \(p^2\ge4q\) thì cần chứng minh:

$$2\,p \left( 19\,{p}^{2}-36\,q \right) r+2\, \left( {p}^{2}-4\,q
 \right) \left( {p}^{4}-2\,{q}^{2} \right) \geqq 0$$

(Hiển nhiên)

Từ 2 TH trên ta thu được điều phải chứng minh.

15 tháng 4 2020

bđt \(\Leftrightarrow\)\(\left(ab+1\right)\left(bc+1\right)\left(ca+1\right)\ge\left(\frac{10}{3}\right)^3abc\) (*) 

đặt \(\left(\sqrt{ab};\sqrt{bc};\sqrt{ca}\right)=\left(x;y;z\right)\)\(\Rightarrow\)\(xyz\le\frac{1}{27}\)

(*) \(\Leftrightarrow\)\(\left(x^2+1\right)\left(y^2+1\right)\left(z^2+1\right)\ge\left(\frac{10}{3}\right)^3xyz\)

\(VT\ge\left(xy+1\right)\left(yz+1\right)\left(zx+1\right)\)

Có \(xy+1\ge10\sqrt[10]{\frac{xy}{9^9}}\)

Tương tự với \(yz+1\)\(;\)\(zx+1\)\(\Rightarrow\)\(VT\ge10^3\sqrt[10]{\frac{\left(xyz\right)^2}{9^{27}}}\)

Ta cần CM \(10^3\sqrt[10]{\frac{\left(xyz\right)^2}{9^{27}}}\ge\frac{10^3}{3^3}xyz\) đúng với \(xyz\le\frac{1}{27}\)

Dấu "=" xảy ra khi \(a=b=c=\frac{1}{3}\)

15 tháng 4 2020

Đặt \(P=\left(a+\frac{1}{b}\right)\left(b+\frac{1}{c}\right)\left(c+\frac{1}{a}\right)\)

Vì a+b+c=1 nên 

\(P=\left(a+\frac{1}{b}\right)\left(b+\frac{1}{c}\right)\left(c+\frac{1}{a}\right)=abc+\frac{1}{abc}+\frac{1}{a}+\frac{1}{b}+\frac{1}{c}+1\)

Từ BĐt Cosi cho 3 số dương ta có:

\(\frac{1}{3}=\frac{a+b+c}{3}\ge\sqrt[3]{abc}\Rightarrow abc\le\frac{1}{27}\)

đặt x=abc thì \(0< x\le\frac{1}{27}\)

do đó: \(x+\frac{1}{x}-27-\frac{1}{27}=\frac{\left(27-x\right)\left(1-27x\right)}{27x}\ge0\)

=> \(x+\frac{1}{x}=abc+\frac{1}{abc}\ge27+\frac{1}{27}=\frac{730}{27}\)

Mặt khác: \(\left(a+b+c\right)\left(\frac{1}{a}+\frac{1}{b}+\frac{1}{c}\right)\ge9\Rightarrow\frac{1}{a}+\frac{1}{b}+\frac{1}{c}\ge9\)

Nên  \(P\ge\frac{730}{27}+10=\frac{1000}{27}=\left(\frac{10}{3}\right)^3\)

Dấu "=" xảy ra khi a=b=c\(=\frac{1}{3}\)

2 tháng 7 2020

Bạn tham khảo tại đây:

Câu hỏi của Trần Hữu Ngọc Minh - Toán lớp 9 - Học toán với OnlineMath

2 tháng 7 2020

Áp dụng BĐT Cosi ta được:

\(\frac{a^3}{\left(1+b\right)\left(1+c\right)}+\frac{1+b}{8}+\frac{1+c}{8}\ge3\sqrt{\frac{a^3\left(1+b\right)\left(1+c\right)}{\left(1+b\right)\left(1+c\right)64}}=\frac{3a}{4}̸\)

Tương tự \(\hept{\begin{cases}\frac{b^3}{\left(1+a\right)\left(1+c\right)}+\frac{1+a}{8}+\frac{1+c}{8}\ge\frac{3b}{4}\\\frac{c^3}{\left(1+a\right)\left(1+b\right)}+\frac{1+a}{8}+\frac{1+b}{8}\ge\frac{3c}{4}\end{cases}}\)

Cộng theo từng vế BĐT trên ta có:

\(\frac{a^3}{\left(1+b\right)\left(1+c\right)}+\frac{b^3}{\left(1+a\right)\left(1+c\right)}+\frac{c^3}{\left(1+a\right)\left(1+b\right)}+\frac{3}{4}\ge\frac{a+b+c}{2}\)

Vì \(a+b+c\ge3\sqrt[3]{abc}=3\)do đó:

\(\frac{a^3}{\left(1+b\right)\left(1+c\right)}+\frac{b^3}{\left(1+a\right)\left(1+c\right)}+\frac{c^3}{\left(1+a\right)\left(1+b\right)}+\frac{3}{4}\ge\frac{3}{2}\)

\(\Leftrightarrow\frac{a^3}{\left(1+b\right)\left(1+c\right)}+\frac{b^3}{\left(1+a\right)\left(1+c\right)}+\frac{c^3}{\left(1+a\right)\left(1+b\right)}\ge\frac{3}{4}\left(đpcm\right)\)

Đẳng thức xảy ra <=> a=b=c

18 tháng 9 2021

Ta có \(a+b+c\ge3\sqrt[3]{abc}=3\)

Áp dụng bđt cosi ta có:

\(\frac{a^3}{\left(b+1\right)\left(c+2\right)}+\frac{b+1}{12}+\frac{c+2}{18}\ge3\sqrt[3]{\frac{a^3}{12.18}}=\frac{a}{2}\)

Làm tương tự

=>\(VT+\left(\frac{a+1}{12}+\frac{a+2}{18}\right)+\left(\frac{b+1}{12}+\frac{b+2}{18}\right)+\left(\frac{c+1}{12}+\frac{c+2}{18}\right)\ge\frac{a+b+c}{2}\)

=> \(VT\ge\frac{13}{36}.\left(a+b+c\right)-\frac{7}{12}\ge\frac{13}{36}.3-\frac{7}{12}=\frac{1}{2}\)(ĐPCM)

21 tháng 9 2021

dấu suy ra thứ 2 phải là lớn hơn hoặc bằng 8(a+b+c)/36-7/12 chứ

Giúp mình với! Mình đang cần gấp. Các bạn làm được bài nào thì giúp đỡ mình nhé! Cảm ơn!Bài 1: Cho các số thực dương a,b,c. Chứng minh rằng:\(\frac{a^2}{\sqrt{\left(2a^2+b^2\right)\left(2a^2+c^2\right)}}+\frac{b^2}{\sqrt{\left(2b^2+c^2\right)\left(2b^2+a^2\right)}}+\frac{c^2}{\sqrt{\left(2c^2+a^2\right)\left(2c^2+b^2\right)}}\le1\).Bài 2: Cho các số thực dương a,b,c,d. Chứng minh...
Đọc tiếp

Giúp mình với! Mình đang cần gấp. Các bạn làm được bài nào thì giúp đỡ mình nhé! Cảm ơn!

Bài 1: Cho các số thực dương a,b,c. Chứng minh rằng:

\(\frac{a^2}{\sqrt{\left(2a^2+b^2\right)\left(2a^2+c^2\right)}}+\frac{b^2}{\sqrt{\left(2b^2+c^2\right)\left(2b^2+a^2\right)}}+\frac{c^2}{\sqrt{\left(2c^2+a^2\right)\left(2c^2+b^2\right)}}\le1\).

Bài 2: Cho các số thực dương a,b,c,d. Chứng minh rằng:

\(\frac{a-b}{a+2b+c}+\frac{b-c}{b+2c+d}+\frac{c-d}{c+2d+a}+\frac{d-a}{d+2a+b}\ge0\).

Bài 3: Cho các số thực dương a,b,c. Chứng minh rằng:

\(\frac{\sqrt{b+c}}{a}+\frac{\sqrt{c+a}}{b}+\frac{\sqrt{a+b}}{c}\ge\frac{4\left(a+b+c\right)}{\sqrt{\left(a+b\right)\left(b+c\right)\left(c+a\right)}}\).

Bài 4:Cho a,b,c>0, a+b+c=3. Chứng minh rằng: 

a)\(\frac{a^3}{a^2+ab+b^2}+\frac{b^3}{b^2+bc+c^2}+\frac{c^3}{c^2+ca+a^2}\ge1\).

b)\(\frac{a^3}{a^2+b^2}+\frac{b^3}{b^2+c^2}+\frac{c^3}{c^2+a^2}\ge\frac{3}{2}\).

c)\(\frac{a+1}{b^2+1}+\frac{b+1}{c^2+1}+\frac{c+1}{a^2+1}\ge3\).

Bài 5: Cho a,b,c >0. Chứng minh rằng:

\(\frac{2a^2+ab}{\left(b+c+\sqrt{ca}\right)^2}+\frac{2b^2+bc}{\left(c+a+\sqrt{ab}\right)^2}+\frac{2c^2+ca}{\left(a+b+\sqrt{bc}\right)^2}\ge1\).

8
21 tháng 10 2019

1) Áp dụng bunhiacopxki ta được \(\sqrt{\left(2a^2+b^2\right)\left(2a^2+c^2\right)}\ge\sqrt{\left(2a^2+bc\right)^2}=2a^2+bc\), tương tự với các mẫu ta được vế trái \(\le\frac{a^2}{2a^2+bc}+\frac{b^2}{2b^2+ac}+\frac{c^2}{2c^2+ab}\le1< =>\)\(1-\frac{bc}{2a^2+bc}+1-\frac{ac}{2b^2+ac}+1-\frac{ab}{2c^2+ab}\le2< =>\)

\(\frac{bc}{2a^2+bc}+\frac{ac}{2b^2+ac}+\frac{ab}{2c^2+ab}\ge1\)<=> \(\frac{b^2c^2}{2a^2bc+b^2c^2}+\frac{a^2c^2}{2b^2ac+a^2c^2}+\frac{a^2b^2}{2c^2ab+a^2b^2}\ge1\)  (1) 

áp dụng (x2 +y2 +z2)(m2+n2+p2\(\ge\left(xm+yn+zp\right)^2\)

(2a2bc +b2c2 + 2b2ac+a2c2 + 2c2ab+a2b2). VT\(\ge\left(bc+ca+ab\right)^2\)   <=> (ab+bc+ca)2. VT \(\ge\left(ab+bc+ca\right)^2< =>VT\ge1\)  ( vậy (1) đúng)

dấu '=' khi a=b=c

21 tháng 10 2019

4b, \(\frac{a^3}{a^2+b^2}+\frac{b^3}{b^2+c^2}+\frac{c^3}{c^2+a^2}=1-\frac{ab^2}{a^2+b^2}+1-\frac{bc^2}{b^2+c^2}+1-\frac{ca^2}{a^2+c^2}\)

\(\ge3-\frac{ab^2}{2ab}-\frac{bc^2}{2bc}-\frac{ca^2}{2ac}=3-\frac{\left(a+b+c\right)}{2}=\frac{3}{2}\)

15 tháng 11 2017

ta có: \(\frac{a}{\left(a+1\right)\left(b+1\right)}+\frac{b}{\left(b+1\right)\left(c+1\right)}+\frac{c}{\left(c+1\right)\left(a+1\right)}.\)

\(\ge3\sqrt[3]{\frac{a.b.c}{\left(a+1\right)^2.\left(b+1\right)^2.\left(c+1\right)^2}}=\frac{3}{\sqrt[3]{\left(a+1\right)^2.\left(b+1\right)^2.\left(c+1\right)^2}}\)    (vì abc=1)     (*)

Mặt khác: \(\left(a+1\right)^2.\left(b+1\right)^2.\left(c+1\right)^2\ge64abc=64=4^3\)   (vì abc=1)

=> \(\sqrt[3]{\left(a+1\right)^2.\left(b+1\right)^2.\left(c+1\right)^2}\ge4\)   (**)

Từ (*), (**)=> đpcm

12 tháng 2 2020

Bạn dưới kia làm ngược dấu thì phải,mà bài này hình như là mũ 3

\(\frac{a^3}{\left(a+1\right)\left(b+1\right)}+\frac{a+1}{8}+\frac{b+1}{8}\ge3\sqrt[3]{\frac{a^3\left(a+1\right)\left(b+1\right)}{64\left(a+1\right)\left(b+1\right)}}=\frac{3a}{4}\)

Tương tự rồi cộng lại:

\(RHS+\frac{2\left(a+b+c\right)+6}{8}\ge\frac{3\left(a+b+c\right)}{4}\)

\(\Leftrightarrow RHS\ge\frac{3}{4}\) tại a=b=c=1